Archivo de la etiqueta: Jordan

Álgebra Lineal II: Unicidad de la forma canónica de Jordan

Por Leonardo Ignacio Martínez Sandoval

Introducción

En la entrada anterior enunciamos el teorema de la forma canónica de Jordan y demostramos la existencia de dicha forma bajo ciertas hipótesis. Como corolario, quedó pensar cuál es la versión para matrices. En esta entrada enunciamos la versión para matrices (totalmente equivalente a la de transformaciones lineales) y nos enfocamos en mostrar la unicidad de la forma canónica de Jordan.

Unicidad de la forma canónica de Jordan

El siguiente teorema es totalmente análogo al enunciado en la entrada anterior. Recuerda que $\leq$ es un orden total fijo de $F$ (en $\mathbb{R}$, es el orden usual).

Teorema. Sea $A$ una matriz $M_n(F)$ cuyo polinomio característico $\chi_A(X)$ se divide en $F$. Entonces, existen únicos valores $\lambda_1\leq \ldots \leq \lambda_n$ en $F$ y únicos enteros $k_1,\ldots,k_d$ tales que \begin{align*} &k_1+k_2+\ldots+k_d = n,\\ &k_1\leq k_2 \leq \ldots \leq k_d,\end{align*} para los cuales $A$ es similar a la siguiente matriz de bloques de Jordan:

$$\begin{pmatrix} J_{\lambda_1,k_1} & 0 & \cdots & 0 \\ 0 & J_{\lambda_2,k_2} & \cdots & 0 \\ \vdots & & \ddots & \vdots \\ 0 & 0 & \cdots & J_{\lambda_d,k_d}\end{pmatrix}.$$

Usaremos esta versión para demostrar la unicidad, lo cual también implicará la unicidad para la versión de transformaciones lineales.

Mediante la demostración de existencia de la entrada anterior, llegamos a que si el polinomio característico de $A$ es

$$\chi_A(X)=(X-\lambda_1)^{m_1}(X-\lambda_2)^{m_2}\cdots(X-\lambda_r)^{m_r},$$

entonces $A$ es similar a una matriz conformada por matrices de bloques de Jordan $J_1,J_2,\ldots,J_r$, en donde cada $J_i$ es de tamaño $m_i$ y de bloques de Jordan de eigenvalor $\lambda_i$.

Si $A$ fuera similar a otra matriz $K$ de bloques de Jordan, podríamos agrupar por eigenvalores de los bloques $\kappa_1< \ldots < \kappa_s$ en matrices de bloques de Jordan tamaños $o_1,\ldots,o_s$, digamos $K_1,\ldots,K_s$. El polinomio característico de $K$ sería entonces

$$\chi_{K}(X)=(X-\kappa_1)^{o_1}(X-\kappa_2)^{o_2}\cdots(X-\kappa_s)^{o_s}.$$

Pero $K$ es similar a $A$, y entonces deben tener el mismo polinomio característico, así que conciden en raíces y multiplicidad. Esto demuestra que $r=s$ y como los $\lambda_i$ y los $\kappa_i$ están ordenados, también demuestra las igualdades $\lambda_i=\kappa_i$ y $m_i=o_i$ para todo $i\in\{1,\ldots,r\}.$

Sólo nos queda argumentar la igualdad entre cada $J_i$ y $K_i$ para $i\in\{1,\ldots,r\}$. Pero ambas una forma canónica de Jordan para la transformación nilpotente que se obtiene de restringir $T_{A-\lambda_i I}$ a $\ker(T_{A-\lambda_i I}^{m_i})$. Por la unicidad que demostramos para la forma canónica de Jordan para transformaciones nilpotentes, concluimos que $J_i=K_i$. Esto termina la demostración de la unicidad de la forma canónica de Jordan.

$\square$

Una receta para encontrar la forma canónica de Jordan

Ya con el teorema demostrado, ¿cómo juntamos todas las ideas para encontrar la forma canónica de Jordan de una matriz $A$ en $M_n(F)$ cuyo polinomio característico se divida en $F$? Podemos proceder como sigue.

  1. Encontramos el polinomio característico $\chi_A(X)$ y su factorización, digamos $$\chi_A(X)=(X-\lambda_1)^{m_1}(X-\lambda_2)^{m_2}\cdots(X-\lambda_r)^{m_r}.$$
  2. Nos enfocamos en encontrar las matrices de bloque de Jordan $J_i$ para cada eigenvalor $\lambda_i$. Sabemos que la matriz $J_i$ será de tamaño $m_i$.
  3. Para saber exactamente cuál matriz de bloques de Jordan es $J_i$, pensaremos en que tiene $b_1,b_2,\ldots,b_{m_i}$ bloques de Jordan de eigenvalor $\lambda_i$ de tamaños $1,2, \ldots,m_i$. Consideramos la matriz $A_i=A-\lambda_i I$. Los $b_1,\ldots,b_{m_i}$ son la solución al siguiente sistema de ecuaciones en las variables $x_1,\ldots,x_{m_i}$.
    \begin{align*}
    m_i&= 1\cdot x_1 + 2\cdot x_2 + 3 \cdot x_3 + \ldots + m_i \cdot x_{m_i}\\
    m_i-n+\text{rango}(A_i-\lambda_i I)&=0\cdot x_1 + 1\cdot x_2 + 2 \cdot x_3 + \ldots + (m_i-1) \cdot x_{m_i}\\
    m_i-n+\text{rango}({A_i-\lambda_i I}^2)&= 0 \cdot x_1 + 0 \cdot x_2 + 1 \cdot x_3 + \ldots + (m_i-2)\cdot x_{m_i}\\
    m_i-n+\text{rango}({A_i-\lambda_i I}^3)&= 0 \cdot x_1 + 0 \cdot x_2 + 0 \cdot x_3 + \ldots + (m_i-3)\cdot x_{m_i}\\
    &\vdots\\
    m_i-n+\text{rango}({A_i-\lambda_i I}^{m_i-1})&= 0\cdot x_1 + 0 \cdot x_2 + 0 \cdot x_3 + \ldots + 1 \cdot x_{m_i}.
    \end{align*}
  4. Juntamos todos los $J_i$ en una misma matriz y los ordenamos apropiadamente.

El paso número $3$ está motivado por lo que sabemos de las matrices nilpotentes, y es bueno que pienses por qué se estudia específicamente ese sistema de ecuaciones para cada eigenvalor $\lambda_i$ y multiplicidad $m_i$.

Ejemplo de obtener la forma canónica de Jordan

Veamos un ejemplo del procedimiento descrito en la sección anterior.

Ejemplo. Encontraremos la forma canónica de Jordan de la siguiente matriz: $$A=\begin{pmatrix}-226 & -10 & -246 & 39 & 246\\234 & 23 & 236 & -46 & -236\\-198 & -20 & -192 & 41 & 195\\-93 & 10 & -122 & 10 & 122\\-385 & -30 & -393 & 74 & 396\end{pmatrix}.$$

Con herramientas computacionales, podemos darnos cuenta de que el polinomio característico de esta matriz es $$\chi_A(X)=X^{5} – 11 X^{4} + 46 X^{3} – 90 X^{2} + 81 X- 27.$$

Este polinomio se puede factorizar como $$(X-1)^2(X-3)^3.$$ Así, la submatriz de bloques de Jordan $J_1$ de eigenvalor $1$ tendrá tamaño $2$ y la $J_3$ de eigenvalor $3$ tendrá tamaño $3$. Pero, ¿de qué tamaño son cada uno de los bloques de Jordan en cada una de estas matrices?

Para respondernos esto para $J_1$, notamos que sus bloques son de tamaño $1$ y $2$ solamente. Si hay $b_1$ bloques de tamaño $1$ y $b_2$ bloques de tamaño $2$, por la teoría desarrollada arriba tendremos:

\begin{align*}
b_1+2b_2&=2\\
b_2&=2-5+\text{rango}(A-I)=2-5+4=1.
\end{align*}

El rango de $A-I$ lo obtuvimos computacionalmente, pero recuerda que también puede ser obtenido con reducción gaussiana. Resolviendo el sistema, $b_2=1$ y entonces $b_1=0$. Concluimos que en $J_1$ hay un bloque de Jordan de tamaño $2$.

Para $J_3$, reciclemos las variables $b_i$ (para no introducir nuevas). Los bloques pueden ser de tamaño $1,2,3$. Supongamos que de estos tamaños respectivamente hay $b_1,b_2,b_3$ bloques. Los $b_i$ cumplen:

\begin{align*}
b_1+2b_2+3b_3&=3\\
b_2+2b_3&=3-5+\text{rango}(A-3I)=3-5+3=1\\
b_3&=3-5+\text{rango}((A-3I)^2)=3-5+2=0.
\end{align*}

Así, $b_3=0$, y en consecuencia $b_2=1$ y entonces $b_1=1$. Concluimos que $J_3$ tiene un bloque de tamaño $1$ y uno de tamaño $3$. Por lo tanto, la forma canónica de Jordan de $A$ es:

$$\begin{pmatrix} J_1 & 0 \\ 0 & J_3 \end{pmatrix} = \begin{pmatrix} J_{1,2} & 0 & 0 \\ 0 & J_{3,1} & 0 \\ 0 & 0 & J_{3,2} \end{pmatrix} = \begin{pmatrix}1 & 1 & 0 & 0 & 0\\0 & 1 & 0 & 0 & 0\\0 & 0 & 3 & 0 & 0\\0 & 0 & 0 & 3 & 1\\0 & 0 & 0 & 0 & 3\end{pmatrix}$$

$\triangle$

Otro problema sobre forma canónica de Jordan

La receta anterior funciona en general y da la forma canónica de Jordan. Esto es algo que probablemente en la práctica en aplicaciones no tendrás que hacer manualmente nunca, pues hay herramientas computacionales que te pueden ayudar. Sin embargo, es importante entender con profundidad el teorema y la receta de manera teórica, pues hay problemas conceptuales en los que no podrás usar herramientas computacionales. A continuación veremos un ejemplo.

Problema. Sea $A$ una matriz en $M_6(\mathbb{R})$ con polinomio característico $$\chi_A(X)=X^6-2X^4+X^2.$$

  • ¿Cuántas posibilidades hay para la forma canónica de Jordan de $A$?
  • Demuestra que si el rango de $A$ es $5$, entonces $A$ no es diagonalizable.

Solución. Podemos factorizar el polinomio característico de $A$ como sigue:

$$\chi_A(X)=X^2(X+1)^2(X-1)^2.$$

Así, la forma canónica de Jordan está conformada por una matriz de bloques de Jordan $J_0$ de eigenvalor $0$ y tamaño $2$; una $J_1$ de eigenvalor $1$ y tamaño $2$; y una $J_{-1}$ de eigenvalor $-1$ y tamaño $2$.

Cada $J_i$ tiene dos chances: o es un bloque de Jordan de tamaño $2$, o son dos bloques de Jordan de tamaño $1$. Así, en total tenemos $2\cdot 2 \cdot 2=8$ posibilidades.

Si $A$ es de rango $5$, entonces tendríamos en las cuentas de cantidad de bloques $b_1$ y $b_2$ para eigenvalor $0$ que

\begin{align*}
b_1+2b_2&=2\\
b_2&=2-6+\text{rango}(A)=2-6+5=1,
\end{align*}

de donde en $J_0$ tendría $1$ bloque de tamaño $2$ y ninguno de tamaño $1$. Si $A$ fuera diagonalizable, su diagonalización sería una forma canónica de Jordan donde para eigenvalor $0$ se tendrían $2$ bloques de tamaño $1$ y ninguno de tamaño $2$. Así, $A$ tendría dos formas canónicas de Jordan distintas, lo cual es imposible.

$\square$

Más adelante…

Con esta entrada terminamos de demostrar el teorema de la forma canónica de Jordan, uno de los teoremas más bonitos de álgebra lineal. ¿Te das cuenta de todo lo que utilizamos en su demostración? Forma matricial de transformaciones lineales, el teorema de Cayley-Hamilton, polinomio característico, subespacios estables, teoría de dualidad, sistemas de ecuaciones lineales, resultados auxiliares de polinomios, etc. Es un resultado verdaderamente integrador.

En la siguiente entrada, la última del curso, hablaremos de algunas de las consecuencias del teorema de la forma canónica de Jordan. Discutiremos cómo lo podemos utilizar para clasificar a las matrices por similaridad. Veremos una aplicación con respecto a una matriz y su transpuesta. También, esbozaremos un poco de por qué en cierto sentido el resultado no sólo vale para las matrices cuyo polinomio se divide sobre el campo, sino que para cualquier matriz. Con ello terminaremos el curso.

Tarea moral

  1. Calcula la forma canónica de Jordan $J$ de la matriz $$A=\begin{pmatrix} 1 & 0 & -3 \\ 1 & -1 & -6 \\ -1 & 2 & 5 \end{pmatrix}.$$ Además de encontrar $J$, encuentra de manera explícita una matriz invertible $P$ tal que $A=P^{-1}JP$.
  2. Calcula la forma canónica de Jordan de la matriz $$\begin{pmatrix} 1 & 1 & 0 & 0 \\ 0 & 1 & 2 & 0 \\ 0 & 0 & 1 & 0 \\ 0 & 0 & 0 & 2 \end{pmatrix}$$
  3. Explica y demuestra cómo obtener lo siguiente para una matriz de bloques de Jordan:
    • Su polinomio característico.
    • Su polinomio mínimo.
    • Su determinante.
    • Su traza.
    • Sus eigenespacios.
  4. Justifica con más detalle por qué la receta que se propone para calcular la forma canónica de Jordan en efecto funciona. Necesitarás varios de los argumentos que dimos en la entrada anterior.
  5. Demuestra que una matriz $A\in M_n(F)$ para la cual su polinomio característico se divide en $F$ es diagonalizable si y sólo si cada bloque de cada matriz de bloques de la forma canónica de Jordan tiene tamaño $1$.

Entradas relacionadas

Agradecimientos

Trabajo realizado con el apoyo del Programa UNAM-DGAPA-PAPIME PE109323 «Hacia una modalidad a distancia de la Licenciatura en Matemáticas de la FC-UNAM – Etapa 3»

Álgebra Lineal II: Existencia de la forma canónica de Jordan

Por Leonardo Ignacio Martínez Sandoval

Introducción

En las entradas anteriores demostramos que para cualquier matriz nilpotente existe (y es única) una matriz similar muy sencilla, hecha por lo que llamamos bloques de Jordan de eigenvalor cero. Lo que haremos ahora es mostrar una versión análoga de este resultado para una familia mucho más grande de matrices. De hecho, en cierto sentido tendremos un resultado análogo para todas las matrices.

Pensando en ello, lo que haremos en esta entrada es lo siguiente. Primero, generalizaremos nuestra noción de bloques de Jordan para contemplar cualquier eigenvalor. Estudiaremos un poco de los bloques de Jordan. Luego, enunciaremos el teorema que esperamos probar. Finalmente, daremos el primer paso hacia su demostración. En la siguiente entrada terminaremos la demostración y hablaremos de aspectos prácticos para encontrar formas canónicas de Jordan.

Enunciado del teorema de la forma canónica de Jordan

A continuación definimos a los bloques de Jordan para cualquier eigenvalor y tamaño.

Definición. Sea $F$ un campo. El bloque de Jordan de eigenvalor $\lambda$ y tamaño $k$ es la matriz $J_{\lambda,k}$ en $M_k(F)$ cuyas entradas son todas $\lambda$, a excepción de las que están inmediatamente arriba de la diagonal superior, las cuales son unos. En símbolos, $J_{\lambda,k}=[a_{ij}]$ con $$a_{ij}=\begin{cases} 1 & \text{si $j=i+1$}\\ \lambda & \text{si $i=j$} \\ 0 & \text{en otro caso.} \end{cases}$$

También podemos expresarlo de la siguiente manera:

$$J_{\lambda,k}=\begin{pmatrix} \lambda & 1 & 0 & \cdots & 0 & 0 \\ 0 & \lambda & 1 & \cdots & 0 & 0 \\ 0 & 0 & \lambda & \cdots & 0 & 0 \\ & \vdots & & \ddots & & \vdots \\ 0 & 0 & 0 & \cdots & \lambda & 1 \\ 0 & 0 & 0 & \cdots & 0 & \lambda \end{pmatrix},$$ en donde estamos pensando que la matriz es de $k\times k$.

Una última manera en la que nos convendrá pensar a $J_{\lambda,k}$ es en términos de los bloques de Jordan de eigenvalor cero: $J_{\lambda,k}=\lambda I_k + J_{0,k}$.

Definición. Una matriz de bloques de Jordan en $M_n(F)$ es una matriz diagonal por bloques en la que cada bloque en la diagonal es un bloque de Jordan.

Lo que nos gustaría demostrar es el siguiente resultado. En él, piensa en $\leq$ como algún orden total fijo de $F$ (para $\mathbb{R}$ es el orden usual, pero otros campos no necesariamente tienen un orden natural asociado).

Teorema. Sea $V$ un espacio vectorial de dimensión finita $n$ sobre el campo $F$ y $T:V\to V$ una transformación lineal tal que $\chi_T(X)$ se divide sobre $F$. Entonces, existen únicos valores $\lambda_1\leq \ldots \leq \lambda_n$ en $F$ y únicos enteros $k_1,\ldots,k_d$ tales que \begin{align*} &k_1+k_2+\ldots+k_d = n,\\ &k_1\leq k_2 \leq \ldots \leq k_d,\end{align*} para los cuales existe una base de $V$ en la cual $T$ tiene como forma matricial a la siguiente matriz de bloques de Jordan:

$$\begin{pmatrix} J_{\lambda_1,k_1} & 0 & \cdots & 0 \\ 0 & J_{\lambda_2,k_2} & \cdots & 0 \\ \vdots & & \ddots & \vdots \\ 0 & 0 & \cdots & J_{\lambda_d,k_d}\end{pmatrix}.$$

Por supuesto, este teorema también tiene una versión matricial, la cuál tendrás que pensar cómo escribir.

Un teorema de descomposición de kernels

Ya tenemos uno de los ingredientes que necesitamos para dar la demostración de la existencia de la forma canónica de Jordan: su existencia para las transformaciones nilpotentes. Otro de los ingredientes que usaremos es el teorema de Cayley-Hamilton. El tercer ingrediente es un resultado de descoposición de kernels de transformaciones evaluadas en polinomios.

Proposición. Sea $V$ un espacio vectorial sobre $F$. Sea $T:V\to V$ una transformación lineal. Y sean $P_1(X),\ldots,P_r(X)$ polinomios en $F[x]$ cuyo máximo común divisor de cualesquiera dos de ellos es el polinomio $1$. Entonces, $$\ker((P_1P_2\cdots P_r)(T))=\bigoplus_{i=1}^r \ker(P_i(T)).$$

Demostración. Para cada $i\in \{1,2,\ldots,r\}$ consideraremos a $Q_i(X)$ como el polinomio que se obtiene de multiplicar a todos los polinomios dados, excepto $P_i(X)$. Y por comodidad, escribiremos $P(X)=(P_1\cdots P_r)(X)$. Notemos que entonces $P(X)=(Q_iP_i)(X)$ para cualquier $i\in\{1,2,\ldots,r\}$.

Primero probaremos un resultado polinomial auxiliar. Veremos que $Q_1(X),\ldots,Q_r(X)$ tienen como máximo común divisor al polinomio $1$. En caso de no ser así, un polinomio $D(X)$ no constante dividiría a todos ellos. Sin pérdida de generalidad, $D$ es irreducible (tomando, por ejemplo $D(X)$ de grado mínimo con esta propiedad). Como $D(X)$ es irreducible y divide a $Q_r(X)$, entonces debe dividir a alguno de los factores de $Q_r(X)$, que sin pérdida de generalidad (por ejemplo, reetiquetando), es $P_1(X)$. Pero $D(X)$ también divide a $Q_1(X)$, así que debe dividir a alguno de sus factores $P_2(X),\ldots,P_r(X)$, sin pérdida de generalidad a $P_2(X)$. Pero entonces $D(X)$ divide a $P_1(X)$ y $P_2(X)$, lo cual contradice las hipótesis. Así, $Q_1(X),\ldots,Q_r(X)$ tienen como máximo común divisor al polinomio $1$. Por el lema de Bézout para polinomios (ver tarea moral), existen entonces polinomios $R_1(X),\ldots,R_r(X)$ tales que

\begin{equation}
\label{eq:bezout}(R_1Q_1 + R_2Q_2 + \ldots + R_rQ_r)(X)=1.
\end{equation}

Estamos listos para pasar a los argumentos de álgebra lineal. Veamos primero que cualquier elemento en la suma de la derecha está en el kernel de $P(T)$. Tomemos $v=v_1+\ldots+v_r$ con $v_i\in \ker(P_i(T))$. Al aplicar $P$ obtenemos

\begin{align*}
P(v)&=P(v_1)+\ldots+P(v_r)\\
&=Q_1(P_1(v_1))+\ldots+Q_r(P_r(v_r))\\
&=0+\ldots+0=0.
\end{align*}

Esto muestra que $v\in \ker(P(T))$, de donde se obtiene la primera contención que nos interesa.

Veamos ahora la segunda contención, que $\ker(P(T))=\bigoplus_{i=1}^r \ker(P_i(T))$. Tomemos $v\in \ker(P(T))$. Al aplicar \eqref{eq:bezout} en $T$ y evaluar en $v$ obtenemos que

\begin{align*}
v&=\text{Id}(v)=(1)(T)(v)\\
&=(R_1Q_1 + R_2Q_2 + \ldots + R_rQ_r)(T)(v)\\
&=(R_1Q_1)(T)(v)+\ldots+(R_rQ_r)(T)(v).
\end{align*}

Pero esto justo expresa a $v$ como elemento de $\ker(P_i(T))$ pues para cada $i$ tenemos

\begin{align*}
P_i(T)((R_iQ_i)(T)(v))&=(P_iR_i Q_i )(T)(v)\\
&=(R_i Q_i P_i)(T)(v)\\
&=R_i(T)P(T)(v)\\
&=R_i(0)=0,
\end{align*}

de modo que expresamos a $v$ como suma de vectores en $\ker(P_1(T)),\ldots,\ker(P_r(T))$.

Ya demostramos la igualdad de conjuntos, pero recordemos que en la igualdad de suma directa hay otra cosa que hay que probar: que el cero tiene una forma única de expresarse como suma de elementos de cada subespacio (aquella en donde cada elemento es cero). Supongamos entonces que $$0=v_1+\ldots+v_r$$ con $v_i\in \ker(P_i(T))$ para cada $i$. Si aplicamos $Q_i$ en esta igualdad, como tiene todos los factores $P_j$ con $j\neq i$ obtenemos $$0=Q_i(0)=Q_i(v_i).$$

Por otro lado, al aplicar nuevamente \eqref{eq:bezout} en $T$ y evaluar en $v_i$

\begin{align*}
v_i&=\text{Id}(v_i)=(1)(T)(v_i)\\
&=(R_1Q_1 + R_2Q_2 + \ldots + R_rQ_r)(T)(v_i)\\
&=(R_1Q_1)(T)(v_1)+\ldots+(R_rQ_r)(T)(v_i)\\
&=(R_iQ_i)(T)(v_i)\\
&=0.
\end{align*}

De esta forma, en efecto tenemos que los espacios están en posición de suma directa, que era lo último que nos faltaba verificar.

$\square$

Existencia de la forma canónica de Jordan

Estamos listos para demostrar la existencia de la forma canónica de Jordan. Supongamos que $V$ es un espacio vectorial de dimensión finita $n$ sobre $F$ y que $T:V\to V$ es una transformación lineal cuyo polinomio característico se divide en $F[x]$. Sabemos entonces que es de la siguiente forma:

$$\chi_T(X)=(X-\lambda_1)^{m_1}(X-\lambda_2)^{m_2}\cdots(X-\lambda_r)^{m_r},$$

donde $\lambda_1,\ldots,\lambda_r$ son eigenvalores distintos de $T$ y $m_1,\ldots,m_r$ son las multiplicidades algebraicas respectivas de estos eigenvalores como raíces de $\chi_T(X)$.

Por el teorema de Cayley-Hamilton, sabemos que $\chi_T(T)=0$, de modo que $\ker(\chi_T(T))=V$. Por la proposición de descomposición de la sección anterior aplicada a los polinomios $P_i(X)=(X-\lambda_i)^{m_i}$ (verifica que son primos relativos dos a dos) para $i\in\{1,\ldots,r\}$ tenemos entonces que $$V=\bigoplus_{i=1}^r \ker((T-\lambda_i \text{id})^{m_i}).$$

Pero, ¿cómo es la transformación $T-\lambda_i \text{id}$ restringida a cada $\ker((T-\lambda_i \text{id})^{m_i})$? ¡Es nilpotente! Precisamente por construcción, $(T-\lambda_i \text{id})^{m_i}$ se anula totalmente en este kernel. Así, por la existencia de la forma canónica de Jordan para matrices nilpotentes, hay una base $\beta_i$ para cada $\ker((T-\lambda_i \text{id})^{m_i})$ tal que $T-\lambda_i \text{id}$ restringida a ese kernel tiene como forma matricial una matriz $J_i$ de bloques de Jordan de eigenvalor cero. Pero entonces $T$ (restringida a dicho kernel) tiene como forma matricial a $J_i+\lambda_i I_{m_i}$, que es una matriz de bloques de Jordan de eigenvalor $\lambda$.

Con esto terminamos: como $V$ es la suma directa de todos esos kernel, la unión de bases $\beta_1,\ldots,\beta_r$ es una base para la cual $T$ tiene como forma matricial a una matriz de bloques de Jordan.

$\square$

Más adelante…

Hemos demostrado la existencia de la forma canónica de Jordan, pero aún nos falta demostrar su unicidad. Además de esto, también necesitaremos un mejor procedimiento para encontrarla. Haremos eso en la siguiente entrada.

Tarea moral

  1. Enuncia el teorema de la forma canónica de Jordan versión matrices.
  2. Investiga más sobre el lema de Bézout para polinomios y cómo se demuestra. Después de esto, expresa al polinomio $1$ como combinación lineal de los polinomios $x^2-1, x^3+1, x^2+5x+4$.
  3. Verifica que los polinomios $P_i(X)=(X-\lambda_i)^{k_i}$ de la demostración de la existencia de la forma canónica de Jordan cumplen las hipótesis de la proposición de descomposición de kernels.
  4. Sea $F$ un campo y $r,s$ elementos en $F$. Sea $n$ un entero. Demuestra que los bloques de Jordan $J_{r,n}$ y $J_{s,n}$ en $M_n(F)$ conmutan.
  5. Siguiendo las ideas de la demostración de existencia, encuentra la forma canónica de Jordan de la matriz $$\begin{pmatrix} 1 & 1 & 1 & 1 \\ 0 & 1 & 1 & 1 \\ 0 & 0 & 2 & 2 \\ 0 & 0 & 0 & 2 \end{pmatrix}.$$

Entradas relacionadas

Agradecimientos

Trabajo realizado con el apoyo del Programa UNAM-DGAPA-PAPIME PE109323 «Hacia una modalidad a distancia de la Licenciatura en Matemáticas de la FC-UNAM – Etapa 3»

Álgebra Lineal II: Unicidad de la forma de Jordan para nilpotentes

Por Leonardo Ignacio Martínez Sandoval

Introducción

En la entrada anterior enunciamos el teorema de la forma canónica de Jordan para matrices nilpotentes. Demostramos una parte: la existencia de la forma canónica de Jordan. Para ello, nos enfocamos en el teorema en su versión en términos de transformaciones lineales. En esta entrada nos enfocaremos en demostrar la unicidad de la forma canónica de Jordan. Curiosamente, en este caso será un poco más cómodo trabajar con la forma matricial del teorema. Para recordar lo que queremos probar, volvemos a poner el enunciado del teorema a continuación. Lo que buscamos es ver que los enteros $k_1,\ldots, k_d$ que menciona el teorema son únicos.

Teorema. Sea $A$ una matriz nilpotente en $M_n(F)$. Entonces existen únicos enteros $k_1,\ldots,k_d$ tales que \begin{align*} &k_1+k_2+\ldots+k_d = n,\\ &k_1\leq k_2 \leq \ldots \leq k_d,\end{align*} y para los cuales $A$ es similar a la siguiente matriz de bloques: $$\begin{pmatrix} J_{0,k_1} & 0 & \cdots & 0 \\ 0 & J_{0,k_2} & \cdots & 0 \\ \vdots & & \ddots & \vdots \\ 0 & 0 & \cdots & J_{0,k_d}\end{pmatrix}.$$

Nuestra estrategia para mostrar la unicidad será el estudio del rango de las potencias de $A$. Si $A$ es similar una matriz en forma canónica $J$, entonces existe $P$ invertible tal que $A=P^{-1}JP$, de donde se puede mostrar indutivamente que $A^k=P^{-1}J^kP$, mostrando que $A^k$ y $J^k$ son similares. Además, sabemos por teoría anterior que matrices similares tienen el mismo rango. De modo que si $A$ es similar a $J$ entonces todas las potencias de $A$ tienen el mismo rango que todas las potencias de $J$. Con esta idea en mente estudiaremos cómo es el rango de matrices de bloques de Jordan de eigenvalor cero.

Rango de potencias de bloques de Jordan

Claramente el rango del bloque de Jordan $J_{0,n}$ es $n-1$, pues ya está en forma escalonada reducida y tiene $n-1$ vectores distintos de cero. El siguiente resultado generaliza esta observación.

Proposición. Sea $n$ un entero positivo, $F$ un campo y $J_{0,n}$ el bloque de Jordan de eigenvalor $0$ y tamaño $n$ en $M_n(F)$. Para $k=1,\ldots,n$ se tiene que el rango de $J_{0,n}^k$ es igual a $n-k$. Para valores de $k$ más grandes, el rango es igual a cero.

Demostración. Si $e_1,\ldots,e_n$ es la base canónica de $F^n$, tenemos que $J_{0,n}e_i=e_{i-1}$ para $i=2,\ldots,n$ y $J_{0,n}e_1=0$. De manera intuitiva, la multiplicación matricial por $J_{0,n}$ va «desplazando los elementos de la base $e_1,\ldots,e_n$ a la izquierda, hasta sacarlos». De este modo, $J_{0,n}^k$ para $k=1,\ldots,n$ hace lo siguiente:

$$J_{0,n}^k e_i=\begin{cases} 0 & \text{para $k\geq i$}\\ e_{i-k} & \text{para $k\leq i-1$.}\end{cases}$$

Así, $J_{0,n}^k$ manda a la base $e_1,\ldots,e_n$ a los vectores $e_1,\ldots,e_{n-k}$ y a $k$ copias del vector cero. Como los primeros son $n-k$ vectores linealmente independientes, obtenemos que el rango de $J_{0,n}^k$ es $n-k$.

Para valores de $k$ más grandes la potencia se hace la matriz cero, así que su rango es cero.

$\square$

Rango de potencias de matrices de bloques de Jordan

¿Qué sucede si ahora estudiamos el rango de las potencias de una matriz de bloques de Jordan? Consideremos, por ejemplo, la siguiente matriz, en donde $k_1,\ldots,k_d$ son enteros positivos de suma $n$ y con $k_1\leq \ldots \leq k_d$:

$$J=\begin{pmatrix} J_{0,k_1} & 0 & \cdots & 0 \\ 0 & J_{0,k_2} & \cdots & 0 \\ \vdots & & \ddots & \vdots \\ 0 & 0 & \cdots & J_{0,k_d}\end{pmatrix}.$$

Por un lado, es sencillo elevar esta matriz a potencias, pues simplemente los bloques se elevan a las potencias correspondientes. En símbolos:

$$J^r=\begin{pmatrix} J_{0,k_1}^r& 0 & \cdots & 0 \\ 0 & J_{0,k_2}^r& \cdots & 0 \\ \vdots & & \ddots & \vdots \\ 0 & 0 & \cdots & J_{0,k_d}^r\end{pmatrix}.$$

¿Cuál es el rango de esta potencia? Nos conviene cambiar un poco de notación. En vez de considerar a los $k_i$ por separado, los agruparemos de acuerdo a su valor, que puede ir de $1$ a $n$. Así, para cada $j=1,\ldots,n$ definimos $m_j$ como la cantidad de valores $k_i$ iguales a $j$. Bajo esta notación, la igualdad $k_1+\ldots+k_d=n$ se puede reescribir como $$m_1+2m_2+3m_3+\ldots+nm_n=n.$$

Una primera observación es que el rango de $J$ es simplemente la suma de los rangos de cada una de las $J_{0,k_i}$. Cada una de éstas contribuye con rango $k_i-1$. Así, en términos de las $m_j$ tenemos lo siguiente:

\begin{align*}
\text{rango}(J)&=\sum_{i=1}^d (k_i-1)\\
&=\sum_{j=1}^n (j-1) m_j \\
&=0\cdot m_1 + 1\cdot m_2 + 2 \cdot m_3 + \ldots + (n-1) \cdot m_n.
\end{align*}

De manera similar,

\begin{align*}
\text{rango}(J^r)&=\sum_{i=1}^d \text{rango}(J_{0,k_i}^r)\\
&=\sum_{j=1}^n m_j \text{rango}(J_{0,j}^r).
\end{align*}

El término $\text{rango}(J_{0,j}^r)$ lo podemos calcular con la proposición de la sección anterior, cuidando la restricción entre el tamaño y las potencias que queremos. De aquí y de la restricción original para la las $m_j$ salen todas las siguientes igualdades:

\begin{align*}
n&= 1\cdot m_1 + 2\cdot m_2 + 3 \cdot m_3 + \ldots + n \cdot m_n\\
\text{rango}(J)&=0\cdot m_1 + 1\cdot m_2 + 2 \cdot m_3 + \ldots + (n-1) \cdot m_n\\
\text{rango}(J^2)&= 0 \cdot m_1 + 0 \cdot m_2 + 1 \cdot m_3 + \ldots + (n-2)\cdot m_n\\
\text{rango}(J^3)&= 0 \cdot m_1 + 0 \cdot m_2 + 0 \cdot m_3 + \ldots + (n-3)\cdot m_n\\
&\vdots\\
\text{rango}(J^{n-1})&= 0\cdot m_1 + 0 \cdot m_2 + 0 \cdot m_3 + \ldots + 1 \cdot m_n.
\end{align*}

A partir de aquí el rango de $J^n$ es $0$. Esto nos da una manera de entender con mucha precisión el rango de cualquier potencia de una matriz diagonal por bloques hecha con bloques de Jordan.

Unicidad de la forma canónica de Jordan

Estamos listos para justificar la unicidad de la forma canónica de Jordan. Una matriz diagonal por bloques hecha por bloques de Jordan queda totalmente determinada por los valores de $m_j$ de la sección anterior. Supongamos que $A$ tiene como forma canónica de Jordan tanto a una matriz $J$ con valores $m_j$, como a otra matriz $J’$ con valores $m_j’$.

Como dos matrices similares cumplen que sus potencias son todas del mismo rango, entonces para cualquier $r$ de $1$ a $n-1$ se cumple que $$\text{rango}(J^r)=\text{rango}(A^r)=\text{rango}(J’^r).$$ Así, tanto $(m_1,\ldots,m_n)$ como $({m_1}’,\ldots,{m_n}’)$ son soluciones al siguiente sistema de ecuaciones en variables $x_1,\ldots,x_n$.

\begin{align*}
n&= 1\cdot x_1 + 2\cdot x_2 + 3 \cdot x_3 + \ldots + n \cdot x_n\\
\text{rango}(A)&=0\cdot x_1 + 1\cdot x_2 + 2 \cdot x_3 + \ldots + (n-1) \cdot x_n\\
\text{rango}(A^2)&= 0 \cdot x_1 + 0 \cdot x_2 + 1 \cdot x_3 + \ldots + (n-2)\cdot x_n\\
\text{rango}(A^3)&= 0 \cdot x_1 + 0 \cdot x_2 + 0 \cdot x_3 + \ldots + (n-3)\cdot x_n\\
&\vdots\\
\text{rango}(A^{n-1})&= 0\cdot x_1 + 0 \cdot x_2 + 0 \cdot x_3 + \ldots + 1 \cdot x_n.
\end{align*}

Pero este es un sistema de $n$ ecuaciones en $n$ variables y con matriz asociada de determinante $1$, así que su solución es única. Esto muestra que $(m_1,\ldots,m_n)=({m_1}’,\ldots,{m_n}’)$. Entonces, en $J$ y $J’$ aparecen la misma cantidad de bloques de cada tamaño. Como además los bloques van de tamaño menor a mayor tanto en $J$ como en $J’$, concluimos que $J=J’$.

Como consecuencia de toda esta discusión, obtenemos de hecho lo siguiente.

Corolario. Dos matrices nilpotentes son semejantes si y sólo si tienen la misma forma canónica de Jordan. Distintas formas canónicas de Jordan dan distintas clases de semejanza.

Una receta para encontrar la forma canónica de Jordan de nilpotentes

La demostración anterior no sólo demuestra la unicidad de la forma canónica de Jordan. Además, nos dice exactamente cómo obtenerla. Para ello:

  1. Calculamos todas las potencias de $A$ hasta $n-1$.
  2. Usando reducción gaussiana (o de otro modo), calculamos el rango de cada una de estas potencias.
  3. Resolvemos el sistema de ecuaciones en variables $x_j$ de la sección anterior.
  4. La forma canónica de Jordan de $A$ tiene $x_j$ bloques de tamaño $j$, que debemos colocar en orden creciente de tamaño.

Ejemplo. Consideremos la siguiente matriz en $M_7(\mathbb{R})$: $$C=\begin{pmatrix}-27 & 266 & 1 & -37 & 135 & -125 & 53\\217 & -1563 & 118 & 33 & -1251 & 1020 & 361\\236 & -1784 & 188 & 16 & -1512 & 1234 & 585\\11 & -10 & -25 & 12 & 28 & -29 & -80\\-159 & 1133 & -114 & -98 & 878 & -690 & -232\\197 & -1409 & 88 & -19 & -1151 & 952 & 348\\-230 & 1605 & -179 & -100 & 1316 & -1031 & -440\end{pmatrix}$$

Sus números son muy complicados, sin embargo, nos podemos auxiliar de herramientas computacionales para encontrar sus potencias. Soprendentemente esta es una matriz nilpotente de índice $3$ pues:

$$C^2=\begin{pmatrix}0 & -10209 & -3403 & -6806 & -6806 & 10209 & 0\\0 & 14691 & 4897 & 9794 & 9794 & -14691 & 0\\0 & 2739 & 913 & 1826 & 1826 & -2739 & 0\\0 & 7221 & 2407 & 4814 & 4814 & -7221 & 0\\0 & -14193 & -4731 & -9462 & -9462 & 14193 & 0\\0 & 10956 & 3652 & 7304 & 7304 & -10956 & 0\\0 & -11952 & -3984 & -7968 & -7968 & 11952 & 0\end{pmatrix}$$

y

$$C^3=\begin{pmatrix}0 & 0 & 0 & 0 & 0 & 0 & 0\\0 & 0 & 0 & 0 & 0 & 0 & 0\\0 & 0 & 0 & 0 & 0 & 0 & 0\\0 & 0 & 0 & 0 & 0 & 0 & 0\\0 & 0 & 0 & 0 & 0 & 0 & 0\\0 & 0 & 0 & 0 & 0 & 0 & 0\\0 & 0 & 0 & 0 & 0 & 0 & 0\end{pmatrix}.$$

Usando reducción gaussiana, o herramientas computacionales, obtenemos que el rango de $C$ es $4$ y que el rango de $C^2$ es $2$. A partir de $k\geq 3$ obtenemos que $\text{rango}(C^k)=\text{rango}(O_7)=0$. Si queremos encontrar la forma canónica de Jordan de $C$, necesitamos entonces resolver el siguiente sistema de ecuaciones, que nos dirá cuántos bloques $x_j$ de tamaño $j$ hay:

\begin{align*}
7&= x_1+2x_2+3x_3+4x_4+5x_5+6x_6+7x_7\\
4&=x_2 + 2x_3 + 3x_4+4x_5+5x_6+6x_7\\
2&= x_3 + 2x_4+3x_5+4x_6+5x_7 \\
0&= x_4+2x_5+3x_6+4x_7\\
0 &= x_5+2x_6+3x_7\\
0&= x_6+2x_7\\
0&= x_7
\end{align*}

Para resolverlo lo mejor es proceder «de abajo hacia arriba». Las últimas cuatro ecuaciones nos dicen que $x_7=x_6=x_5=x_4=0$. Así, el sistema queda un poco más simple, como:

\begin{align*}
7&= x_1+2x_2+3x_3\\
4&=x_2 + 2x_3\\
2&= x_3.
\end{align*}

De la última igualdad, tenemos $x_3=2$, lo que nos dice que la forma canónica de Jordan tendría dos bloques de tamaño $3$. Sustituyendo en la penúltima igualdad obtenemos que $4=x_2+4$, de donde $x_2=0$. Así, no tendremos ningún bloque de tamaño $2$. Finalmente, sustituyendo ambos valores en la primera igualdad obtenemos que $7=x_1+0+6$. De aquí obtenemos $x_1=1$, así que la forma canónica de Jordan tendrá un bloque de tamaño $1$. En resumen, la forma canónica de Jordan es la matriz $$\begin{pmatrix} J_{0,1} & 0 & 0 \\ 0 & J_{0,3} & 0 \\ 0 & 0 & J_{0,3}\end{pmatrix}.$$ Explícitamente, ésta es la siguiente matriz:

$$\begin{pmatrix} 0& 0 & 0 & 0 & 0 & 0 & 0 \\ 0& 0 & 1 & 0 & 0 & 0 & 0 \\ 0& 0 & 0 & 1 & 0 & 0 & 0 \\ 0& 0 & 0 & 0 & 0 & 0 & 0 \\ 0& 0 & 0 & 0 & 0 & 1 & 0 \\ 0& 0 & 0 & 0 & 0 & 0 & 1 \\ 0& 0 & 0 & 0 & 0 & 0 & 0 \end{pmatrix}.$$

Para verla un poco más «como de bloques» la podemos reescribir de la siguiente manera:

$$\left(\begin{array}{c|ccc|ccc} 0& 0 & 0 & 0 & 0 & 0 & 0 \\ \hline 0& 0 & 1 & 0 & 0 & 0 & 0 \\ 0& 0 & 0 & 1 & 0 & 0 & 0 \\ 0& 0 & 0 & 0 & 0 & 0 & 0 \\ \hline 0& 0 & 0 & 0 & 0 & 1 & 0 \\ 0& 0 & 0 & 0 & 0 & 0 & 1 \\ 0& 0 & 0 & 0 & 0 & 0 & 0 \end{array}\right).$$

$\triangle$

Más adelante…

Hemos demostrado la existencia y unicidad de la forma canónica de Jordan para matrices nilpotentes. Este es un resultado interesante por sí mismo. Sin embargo, también es un paso intermedio para un resultado más general. En las siguientes entradas hablaremos de una versión más general del teorema de Jordan, para matrices tales que su polinomio característico se descomponga totalmente en el campo en el que estemos trabajando.

Tarea moral

  1. Considera la siguiente matriz: $$M=\begin{pmatrix}11 & 11 & -11 & -11\\-1 & -1 & 1 & 1\\3 & 3 & -3 & -3\\7 & 7 & -7 & -7\end{pmatrix}.$$
    1. Muestra que $M$ es una matriz nilpotente y determina su índice.
    2. ¿Cuál es la forma canónica de Jordan de $M$?
  2. Describe las posibles formas canónicas de Jordan para una matriz nilpotente $A \in M_{5}(F)$ de índice $2$.
  3. Describe las posibles formas canónicas de Jordan para una matriz nilpotente $A \in M_{7}(F)$ de rango $5$.
  4. Encuentra de manera explícita la inversa de la siguiente matriz en $M_n(\mathbb{R})$ y usa esto para dar de manera explícita la solución al sistema de ecuación en las variables $x_i$ que aparece en la entrada: $$\begin{pmatrix} 1 & 2 & 3 & \cdots & n-1 & n \\ 0 & 1 & 2 & \cdots & n-2 & n-1 \\ 0 & 0 & 1 & \cdots & n-3 & n-2 \\ & \vdots & & \ddots & & \vdots\\ 0 & 0 & 0 & \cdots & 1 & 2 \\ 0 & 0 & 0 & \cdots & 0 & 1\end{pmatrix}.$$
  5. Sea $A$ una matriz nilpotente en $M_n(\mathbb{R})$. Muestra que las matrices $A$ y $5A$ son similares entre sí.

Entradas relacionadas

Agradecimientos

Trabajo realizado con el apoyo del Programa UNAM-DGAPA-PAPIME PE109323 «Hacia una modalidad a distancia de la Licenciatura en Matemáticas de la FC-UNAM – Etapa 3»

Álgebra Lineal II: Otras aplicaciones de formas canónicas de Jordan

Por Elizabeth Chalnique Ríos Alvarado

Introducción

En las notas anteriores desarrollamos teoría interesante acerca de las formas canónicas de Jordan, ahora vamos a ver algunos ejemplos de todo eso.

Ejemplo 1

Considera la matriz $$A = \begin{pmatrix}1 & 0 & 0 & 0 & 2 \\ 0 & 0 & 1 & 0 & 0 \\ 0 & 0 & 0 & 0 & 0 \\ 0 & 1 & 0 & 0 & 0 \\ -1 & 0 & 0 & 0 & -2 \end{pmatrix}$$

Calculamos $\chi_{A}(X)$ expandiendo $det(XI_{5} – A)$ con respecto a la tercera fila y obtenemos (usando de nuevo la expansión respecto a la segunda fila en el nuevo determinante) \begin{align*} \chi_{A}(X) &= X \begin{vmatrix} X-1 & 0 & 0 & -2 \\ 0 & X & 0 & 0 \\ 0 & -1 & X & 0 \\ 1 & 0 & 0 & X+2 \end{vmatrix} \\ &= X^{2} \begin{vmatrix} X-1 & 0 & 2 \\ 0 & X & 0 \\ 1 & 0 & X+2 \end{vmatrix} \\ &= X^{3} \begin{vmatrix} X-1 & -2 \\ 1 & X+2 \end{vmatrix} \\ &= X^{4} (X+1) \end{align*}

El eigenvalor $-1$ tiene multiplicidad algebraica 1, por lo que hay un solo bloque de Jordan asociado con este eigenvalor, de tamaño 1. Ahora, veamos qué pasa con el eigenvalor 0 que tiene multiplicidad algebraica 4. Sea $N_{m}$ el número de bloques de Jordan de tamaño $m$ asociados con ese eigenvalor. Por el Teorema visto en la nota anterior tenemos que $$N_{1} = rango(A^{2}) – 2rango(A) + 5,$$ $$N_{2} = rango(A^{3}) – 2rango(A^{2}) + rango(A)$$ etcétera. Puedes checar fácilmente que $A$ tiene rango 3.

Luego, calculemos $A^{2} = \begin{pmatrix} -1 & 0 & 0 & 0 & -2 \\ 0 & 0 & 0 & 0 & 0 \\ 0 & 0 & 0 & 0 & 0 \\ 0 & 0 & 1 & 0 & 0 \\ 1 & 0 & 0 & 0 & 2 \end{pmatrix}$, $A^{3} = \begin{pmatrix} 1 & 0 & 0 & 0 & 2 \\ 0 & 0 & 0 & 0 & 0 \\ 0 & 0 & 0 & 0 & 0 \\ 0 & 0 & 0 & 0 & 0 \\ -1 & 0 & 0 & 0 & -2 \end{pmatrix}$.

Nota que $A^{2}$ tiene rango 2 (pues una base del generado por sus filas está dada por la primera y cuarta fila) y $A^{3}$ tiene rango 1. De donde, $$N_{1} = 2-2 \cdot 3 + 5 = 1,$$ por lo que hay un bloque de Jordan de tamaño 1 y $$N_{2} = 1-2 \cdot 2 + 3 = 0,$$ entonces no hay un bloque de Jordan de tamaño 2. Dado que la suma de los tamaños de los bloques de Jordan asociados con el eigenvalor 0 es 4, y como ya sabemos que hay un bloque de tamaño 1 y no hay de tamaño 2, deducimos que hay un bloque de tamaño 3 y que la forma canónica de Jordan de $A$ es $$\begin{pmatrix} -1 & 0 & 0 & 0 & 0 \\ 0 & 0 & 0 & 0 & 0 \\ 0 & 0 & 0 & 1& 0 \\ 0 & 0 & 0 & 0 & 1 \\ 0 & 0 & 0 & 0 & 0\end{pmatrix}.$$

Ejemplo 2

Más adelante…

Con esto finalizamos el curso de Álgebra Lineal II, lo que sigue es el maravilloso mundo del Álgebra Moderna.

Tarea moral

A continuación hay algunos ejercicios para que practiques los conceptos vistos en esta entrada. Te será de mucha utilidad intentarlos para entender más la teoría vista.

  1. Usa el Teorema de Jordan para probar que cualquier matriz $A \in M_{n}(\mathbb{C})$ es similar a su transpuesta.
  2. Prueba que si $A \in M_{n}(\mathbb{C})$ es similar a $2A$, entonces $A$ es nilpotente.
  3. Usa el teorema de Jordan para probar que si $A \in M_{n}(\mathbb{C})$ es nilpotente, entonces $A$ es similar a $2A$.

Entradas relacionadas

Agradecimientos

Trabajo realizado con el apoyo del Programa UNAM-DGAPA-PAPIME PE109323 «Hacia una modalidad a distancia de la Licenciatura en Matemáticas de la FC-UNAM – Etapa 3»

Álgebra Lineal II: Clasificación de matrices por similaridad

Por Elizabeth Chalnique Ríos Alvarado

Introducción

En las notas anteriores hemos desarrollado el Teorema de Jordan, y ahora veremos cómo podemos clasificar matrices por similaridad.

Sección

Supongamos que $A$ es una matriz similar a $$\begin{pmatrix} J_{k_{1}}(\lambda_{1}) & 0 & \cdots & 0 \\ 0 & J_{k_{2}}(\lambda_{2}) & \cdots & 0 \\ \vdots & \vdots & \ddots & \vdots \\ 0 & 0 & \cdots & J_{k_{d}}(\lambda_{d}) \end{pmatrix}$$

Entonces el polinomio característico de $A$ es $$\chi_{A}(X) = \prod_{i=1}^{d}\chi_{J_{k_{i}}} (\lambda_{i})(X).$$

Ahora, dado que $J_{n}$ es nilpotente tenemos $\chi_{J_{k_{i}}}(X) = X^{n}$ y así $$\chi_{J_{n}(\lambda)}(X) = (X – \lambda)^{n}.$$

Se sigue que $$\chi_{A}(X) = \prod_{i=1}^{d} (X – \lambda_{i})^{k_{i}}$$ y así necesariamente $\lambda_{1}, \ldots, \lambda_{d}$ son todos eigenvalores de $A$. Nota que no asumimos que $\lambda_{1}, \ldots, \lambda_{d}$ sean distintos a pares, por lo que no podemos concluir de la igualdad anterior que $k_{1}, \ldots, k_{d}$ sean las multiplicidades algebráicas de los eigenvalores de $A$. Esto no es verdad en general: varios bloques de Jordan correspondientes a un dado eigenvalor pueden aparecer. El problema de la unicidad se resuelve completamente por el siguiente:

Teorema: Supongamos que una matriz $A \in M_{n}(F)$ es similar a $$\begin{pmatrix} J_{k_{1}}(\lambda_{1}) & 0 & \cdots & 0 \\ 0 & J_{k_{2}}(\lambda_{2}) & \cdots & 0 \\ \vdots & \vdots & \ddots & \vdots \\ 0 & 0 & \cdots & J_{k_{d}}(\lambda_{d}) \end{pmatrix}$$ para algunos enteros positivos $k_{1}, \ldots, k_{d}$ que suman $n$ y algunas $\lambda_{1}, \ldots, \lambda_{d} \in F$. Entonces

  1. Cada $\lambda_{i}$ es un eigenvalor de $A$.
  2. Para cada eigenvalor $\lambda$ de $A$ y cada entero positivo $m$, el número de bloques de Jordan $J_{m}(\lambda)$ entre $J_{k_{1}}(\lambda_{1}), \ldots, J_{k_{d}}(\lambda_{d})$ is $$N_{m}(\lambda) = rango(A – \lambda I_{n})^{m+1} – 2 rango(A – \lambda I_{n})^{m} + rango(A – \lambda I_{n})^{m-1}$$ y depende sólo en la clase de similaridad de $A$.

Demostración. Ya vimos el inciso 1. La prueba del inciso 2 es muy similar a la solución del Problema __. Más precisamente, sea $B = A – \lambda I_{n}$ y observa que $B^{m}$ es similar a $\begin{pmatrix} (J_{k_{1}}(\lambda_{1}) – \lambda I_{k_{1}})^{m} & 0 & \cdots & 0 \\ 0 & (J_{k_{2}}(\lambda_{2}) – \lambda I_{k_{2}})^{m} & \cdots & 0 \\ \vdots & \vdots & \ddots & \vdots \\ 0 & 0 & \cdots & (J_{k_{d}}(\lambda_{d}) – \lambda I_{k_{d}})^{m}\end{pmatrix}$, por lo que $\displaystyle rango(B^{m}) = \sum_{i=1}^{d} rango(J_{k_{i}} (\lambda_{i}) – \lambda I_{k_{i}})^{m}$.

Ahora, el rango de $(J_{n}(\lambda) – \mu I_{n})^{m}$ es

  • $n$ si $\lambda \neq \mu$, como en este caso $$J_{n}(\lambda) – \mu I_{n} = J_{n} + (\lambda – \mu) I_{n}$$ es invertible,
  • $n-m$ para $\lambda = \mu$ y $m \leq n$, como se sigue del Problema __.
  • 0 para $\lambda = \mu$ y $m > n$, dado que $J^{n}_{n} = O_{n}$.

De ahí, si $N_{m}(\lambda)$ es el número de bloques de Jordan $J_{m}(\lambda)$ entre $J_{k_{1}}(\lambda_{1}), \ldots, J_{k_{d}}(\lambda_{d})$, entonces $$rango(B^{m}) = \sum_{\substack{\lambda_{i} = \lambda \\ k_{i} \geq m}} (k_{i} – m) + \sum_{\lambda_{i} \neq \lambda} k_{i},$$ luego sustrayendo esas igualdades para $m-1$ y $m$ se tiene que $$rango(B^{m-1}) – rango(B^{m}) = \sum_{\substack{\lambda_{i} = \lambda \\ k_{i} \geq m}} 1$$ y finalmente \begin{align*} rango(B^{m-1}) – 2rango(B^{m}) + rango(B^{m+1}) = \\ (rango(B^{m-1}) – rango(B^{m})) – (rango(B^{m}) – rango(B^{m+1})) = \\ \sum_{\substack{\lambda_{i} = \lambda \\ k_{i} = m}} 1 = N_{m}(\lambda) \end{align*} como queríamos.

$\square$

Tarea moral

A continuación hay algunos ejercicios para que practiques los conceptos vistos en esta entrada. Te será de mucha utilidad intentarlos para entender más la teoría vista.

  1. ¿Cuáles son las posibles formas canónicas de Jordan de una matriz cuyo polinomio característico es $(X-1)(X-2)^{2}$?
  2. Considera una matriz $A \in M_{6}(\mathbb{C}) de rango 4 cuyo polinomio mínimo es $X(X-1)(X-2)^{2}$.
    1. ¿Cuáles son los eigenvalores de $A$?
    2. ¿$A$ es diagonalizable?
    3. ¿Cuáles son las posibles formas canónicas de Jordan de $A$?

Más adelante…

En la siguiente nota veremos algunos ejemplos de cómo funciona todo esto.

Entradas relacionadas

Agradecimientos

Trabajo realizado con el apoyo del Programa UNAM-DGAPA-PAPIME PE109323 «Hacia una modalidad a distancia de la Licenciatura en Matemáticas de la FC-UNAM – Etapa 3»